GGIBA003,
Yes, Could Be True EXCEPT = one Cannot Be True/Must Be False
and
Could Be False EXCEPT = one Must Be True
By the time you get to questions #5, you've very likely done the other questions and have some work that can help you answer this one without having to waste time repeating a lot of work. For instance, for question #2, you could have the following hypothetical:
West: SOG
East MUP
In fact, I do have that, based on my answer in this thread:
viewtopic.php?p=92874#p92874
So, when U is at 2, we know all of the following could be true:
G can be at 3 (answer choice (A))
O can be at 2 (answer choice (C))
S can be at 1 (answer choice (E))
Only two answer choices are left, and you can brute-force one of them. If it works, it's wrong, and you pick the other answer. If it doesn't work, you pick it. Either way, one more diagram answers this.
Your first hypothetical for answer choice (A) is only one possibility among many - you're putting U, G, and M into specific theaters. You're showing that that arrangement doesn't work. That only proves that, if you put those three variables are certain times AND certain theaters, it won't work. Why not change the theaters a bit? In fact, as my hypothetical from question #2 shows, we can make answer choice (A) work, but it requires the theaters to be different than what your hypothetical showed. That's fine, because nothing about the local condition for question #5, nor any of the answer choices, determines which theaters things are in. So, ultimately, you're showing that in ONE hypothetical compatible with answer choice (A), the rules are violated. In order to show that answer choice (A) cannot be true at all, you'd have to show that ALL hypotheticals compatible with answer choice (A) violate some rule. Basically, you proved that answer choice (A) doesn't have to be true. You needed to go further - prove it can't be true. Since it can be true, that wouldn't have been possible.
Robert Carroll